Quantcast
  • Register
PhysicsOverflow is a next-generation academic platform for physicists and astronomers, including a community peer review system and a postgraduate-level discussion forum analogous to MathOverflow.

Welcome to PhysicsOverflow! PhysicsOverflow is an open platform for community peer review and graduate-level Physics discussion.

Please help promote PhysicsOverflow ads elsewhere if you like it.

News

PO is now at the Physics Department of Bielefeld University!

New printer friendly PO pages!

Migration to Bielefeld University was successful!

Please vote for this year's PhysicsOverflow ads!

Please do help out in categorising submissions. Submit a paper to PhysicsOverflow!

... see more

Tools for paper authors

Submit paper
Claim Paper Authorship

Tools for SE users

Search User
Reclaim SE Account
Request Account Merger
Nativise imported posts
Claim post (deleted users)
Import SE post

Users whose questions have been imported from Physics Stack Exchange, Theoretical Physics Stack Exchange, or any other Stack Exchange site are kindly requested to reclaim their account and not to register as a new user.

Public \(\beta\) tools

Report a bug with a feature
Request a new functionality
404 page design
Send feedback

Attributions

(propose a free ad)

Site Statistics

205 submissions , 163 unreviewed
5,047 questions , 2,200 unanswered
5,345 answers , 22,709 comments
1,470 users with positive rep
816 active unimported users
More ...

  Evaluating propagator without the epsilon trick

+ 7 like - 0 dislike
2005 views

Consider the KleinGordon equation and its propagator: $$G(x,y) = \frac{1}{(2\pi)^4}\int d^4 p \frac{e^{-i p.(x-y)}}{p^2 - m^2} \; .$$

I'd like to see a method of evaluating explicit form of $G$ which does not involve avoiding singularities by the $\varepsilon$ trick. Can you provide such a method?

This post imported from StackExchange Physics at 2014-07-13 04:38 (UCT), posted by SE-user qoqosz
asked Dec 4, 2012 in Theoretical Physics by qoqosz (35 points) [ no revision ]
retagged Jul 13, 2014

I found some discussions from the book "Quantum Field Theory I Basics in Mathematics and Physics A Bridge between Mathematicians and Physicists".

https://www.amazon.com/Quantum-Field-Theory-Mathematics-Mathematicians/dp/3540347623

It's related with distribution theory in mathematics. This is beyond the scope of ordinary "complex analysis" and is not covered from any complex analysis textbook. The $\epsilon$-trick is not avoidable because the propagator is defined in that way. 

3 Answers

+ 7 like - 0 dislike

Before answering the question more or less directly, I'd like to point out that this is a good question that provides an object lesson and opens a foray into the topics of singular integral equations, analytic continuation and dispersion relations. Here are some references of these more advanced topics: Muskhelishvili, Singular Integral Equations; Courant & Hilbert, Methods of Mathematical Physics, Vol I, Ch 3; Dispersion Theory in High Energy Physics, Queen & Violini; Eden et.al., The Analytic S-matrix. There is also a condensed discussion of `invariant functions' in Schweber, An Intro to Relativistic QFT Ch13d.

The quick answer is that, for $m^2 \in\mathbb{R}$, there's no "shortcut." One must choose a path around the singularities in the denominator. The appropriate choice is governed by the boundary conditions of the problem at hand. The $+i\epsilon$ "trick" (it's not a "trick") simply encodes the boundary conditions relevant for causal propagation of particles and antiparticles in field theory.

We briefly study the analytic form of $G(x-y;m)$ to demonstrate some of these features.

Note, first, that for real values of $p^2$, the singularity in the denominator of the integrand signals the presence of (a) branch point(s). In fact, [Huang, Quantum Field Theory: From Operators to Path Integrals, p29] the Feynman propagator for the scalar field (your equation) may be explicitly evaluated: \begin{align} G(x-y;m) &= \lim_{\epsilon \to 0} \frac{1}{(2 \pi)^4} \int d^4p \, \frac{e^{-ip\cdot(x-y)}}{p^2 - m^2 + i\epsilon} \nonumber \\ &= \left \{ \begin{matrix} -\frac{1}{4 \pi} \delta(s) + \frac{m}{8 \pi \sqrt{s}} H_1^{(1)}(m \sqrt{s}) & \textrm{ if }\, s \geq 0 \\ -\frac{i m}{ 4 \pi^2 \sqrt{-s}} K_1(m \sqrt{-s}) & \textrm{if }\, s < 0. \end{matrix} \right. \end{align} where $s=(x-y)^2$.

The first-order Hankel function of the first kind $H^{(1)}_1$ has a logarithmic branch point at $x=0$; so does the modified Bessel function of the second kind, $K_1$. (Look at the small $x$ behavior of these functions to see this.)

A branch point indicates that the Cauchy-Riemann conditions have broken down at $x=0$ (or $z=x+iy=0$). And the fact that these singularities are logarithmic is an indication that we have an endpoint singularity [eg. Eden et. al., Ch 2.1]. (To see this, consider $m=0$, then the integrand, $p^{-2}$, has a zero at the lower limit of integration in $dp^2$.)

Coming back to the question of boundary conditions, there is a good discussion in Sakurai, Advanced Quantum Mechanics, Ch4.4 [NB: "East Coast" metric]. You can see that for large values of $s>0$ from the above expression that we have an outgoing wave from the asymptotic form of the Hankel function.

Connecting it back to the original references I cited above, the $+i\epsilon$ form is a version of the Plemelj formula [Muskhelishvili]. And the expression for the propagator is a type of Cauchy integral [Musk.; Eden et.al.]. And this notions lead quickly to the topics I mentioned above -- certainly a rich landscape for research.

This post imported from StackExchange Physics at 2014-07-13 04:38 (UCT), posted by SE-user MarkWayne
answered Dec 4, 2012 by MarkWayne (270 points) [ no revision ]
'it's not a "trick"' Agreed. This is a standard procedure from complex analysis. It has the blessing of the same kinds of OCD mathematicians who balk at other shortcuts physicist sometimes take.

This post imported from StackExchange Physics at 2014-07-13 04:38 (UCT), posted by SE-user dmckee
This "standard procedure" was certainly not covered in any of the multiple analysis courses I've taken. I guess we had a different focus... In any event, there is still some disconnect between mathematicians and physicists, where only the latter tell me integrals can depend on path deformations within the domain of analyticity.

This post imported from StackExchange Physics at 2014-07-13 04:38 (UCT), posted by SE-user Chris White
@ChrisWhite Both my undergrad and graduate institution had course called Mathematical Methods in Physics and it was covered in them. The people I know whose programs did not have such a course often haven't seen it. If someone asked me for a text recommendation I'd say Arfkin, but that's just 'cause it was the one I used that I liked more.

This post imported from StackExchange Physics at 2014-07-13 04:38 (UCT), posted by SE-user dmckee
@dmckee Thanks. I'll look into that, as well as the other suggestions. The relevant courses I took/audited were taught by a pure mathematician, an applied mathematician, a dynamics mathematician, and a string theorist. I didn't take the one taught by a run-of-the-mill quantum physicist, alas.

This post imported from StackExchange Physics at 2014-07-13 04:38 (UCT), posted by SE-user Chris White
@ChrisWhite: "only the latter tell me integrals can depend on path deformations within the domain of analyticity" -- then you're talking to someone -- I'm not sure why it's relevant that the "telling" came from someone in a specific profession -- who doesn't understand complex analysis. If you can provide a specific example of this, I'm sure we can clear up the confusion. Sometimes it's difficult to see when a singularity has been 'crossed' etc.

This post imported from StackExchange Physics at 2014-07-13 04:38 (UCT), posted by SE-user MarkWayne
@dmckee: In the states, an established trick is known as a "technique". :)

This post imported from StackExchange Physics at 2014-07-13 04:38 (UCT), posted by SE-user Art Brown
+ 5 like - 0 dislike

Expanding on dmckee's comment:

The $+i\epsilon$-trick has the blessing of OCD mathematicians because it follows directly from a deep fact about the group of spacetime translations: the group $\{e^{-i\langle P,x\rangle/\hbar}| x \in \mathbb{R}^n\}$ of spacetime translations is the boundary of an analytic semigroup $\{e^{-i\langle P,\xi\rangle/\hbar}| x \in \mathbb{C}^n \mbox{ and } Im(\xi) \leq 0\}$.

Many quantities in field theory are expressed in terms of these translations, and frequently these quantities can be computed more easily by analytically continuing from real "Minkowski" time to imagininary "Euclidean" time, where the delicate cancellation of phases becomes the crude suppression of exponential damping. When you use the $+i\epsilon$-trick, what you're really doing is saying that the particular cancellation of phases you want is the one which respects this analyticity. This is precisely what's happening when you use the $+i\epsilon$-trick to evaluate the Klein-Gordon propagator. You've got an integral which does not converge absolutely, and you're picking out a certain resummation which does. The $+i\epsilon$ is not just a trick here; it's really the definition of the quantity you're after.

This post imported from StackExchange Physics at 2014-07-13 04:38 (UCT), posted by SE-user user1504
answered Dec 4, 2012 by user1504 (1,110 points) [ no revision ]
This is very interesting! I didn't know that the group of spacetime translations is the boundary of an analytic semigroup! Where can I read more about this?

This post imported from StackExchange Physics at 2014-07-13 04:38 (UCT), posted by SE-user QuantumDot
You can look in Streater & Wightman's book. But more seriously: just try it out by hand in the 1d case, when there's only time translations. You'll be able to draw a picture of the whole group on the plane.

This post imported from StackExchange Physics at 2014-07-13 04:38 (UCT), posted by SE-user user1504
+ 1 like - 0 dislike

As far as my experience goes, the problem stems from writing the right solution for all reals to the problem: $$ (p-m)G(p)=1. $$ which reads: $$ G(p)=\text{P.v.} \frac {1}{p-m}+c_0\delta(p-m) $$ where $\text{P.v.}$ stands for principal value. The $\delta(\epsilon-\omega)$ function appears as it is the Kernel of $(\omega-\epsilon)$ and $c_0$ is some constant to be fixed. If we now take the Fourier transform we obtain: $$ \int e^{ipt }G(p)=\:\left( i\pi \text{sign}(t)+c_0\right)e^{i m t} $$ $c_0$ must now be fixed according to boundary conditions; for the retarded and advanced Green functions, one has $c_0=\pm i\pi$ and the solution given by the $i \epsilon$ trick is recovered. In my opinion though, it is a rather bad method as it only works when you have poles or first order, as the $\delta$ function can be approached by square integrable functions. If you now are looking for the solution of: $$ (p-m)^kG(p)=1. $$ with k an integer, you now have $$ G(p)=\text{P.v.} \frac {1}{(p-m)^k}+\sum_{j=0}^kc_j\delta^{(j)}(p-m) $$ with $\delta^{(k)}$ the k-th derivative of the delta function. The Fourier transform reads $$ G(t)=\left( i\pi \frac{(i t)^{k-1}}{k-1!}\text{sign}(t)+\sum_{j=0}^kc_j (-it)^j\right)e^{i m t} $$ And again, the $c_j$s are fixed depending on boundary conditions. Yet I don't know any way to recover this solution with the $i\epsilon$ trick.

This post imported from StackExchange Physics at 2014-07-13 04:38 (UCT), posted by SE-user user26784
answered Oct 28, 2013 by user26784 (10 points) [ no revision ]

Your answer

Please use answers only to (at least partly) answer questions. To comment, discuss, or ask for clarification, leave a comment instead.
To mask links under text, please type your text, highlight it, and click the "link" button. You can then enter your link URL.
Please consult the FAQ for as to how to format your post.
This is the answer box; if you want to write a comment instead, please use the 'add comment' button.
Live preview (may slow down editor)   Preview
Your name to display (optional):
Privacy: Your email address will only be used for sending these notifications.
Anti-spam verification:
If you are a human please identify the position of the character covered by the symbol $\varnothing$ in the following word:
p$\hbar$ysicsO$\varnothing$erflow
Then drag the red bullet below over the corresponding character of our banner. When you drop it there, the bullet changes to green (on slow internet connections after a few seconds).
Please complete the anti-spam verification




user contributions licensed under cc by-sa 3.0 with attribution required

Your rights
...